Paradoxes mathématiques

De WikiSbec
Aller à la navigation Aller à la recherche
Cette page fait site WikiSbec de Sébastien Bruneau.
Si vous êtes perdu, vous pouvez retourner à l'accueil en cliquant ici , ou au portail des sites
sbeccompany.fr en cliquant ici
Logo sbec 2019.jpg


Accueil | Mathématiques | Paradoxes



Voici une petite liste de démonstrations mathématiques fausses. On aboutit à chaque fois à un paradoxe mathématique par exemple 1=0. A vous de trouver l'erreur !

1=0

Calculer la somme : 1-1+1-1+1-1+1-1+1-1+1-1+.......... = ?


  • Méthode 1) : <math>\underbrace{(1-1)}_{=0}+\underbrace{(1-1)}_{=0}+\underbrace{(1-1)}_{=0}+\underbrace{(1-1)}_{=0}+\underbrace{(1-1)}_{=0}+\underbrace{(1-1)}_{=0}+..........=0+0+0+0+0+0+..........=0</math>


  • Méthode 2) : <math>1+\underbrace{(-1+1)}_{=0}+\underbrace{(-1+1)}_{=0}+\underbrace{(-1+1)}_{=0}+\underbrace{(-1+1)}_{=0}+\underbrace{(-1+1)}_{=0}+\underbrace{(-1+1)}_{=0}+......=1+0+0+0+0+0+0+......=1</math>


On aboutit rapidement à 1-1+1-1+1-1+1-1+1-1+1-1+.......... =1=0

1=2

  • Partie 1 : <math>1=\ln e=\ln {e} - 0=\ln {e} - \ln {0}=\int^e_1{\frac{dx}{x}}</math>
  • Partie 2 : <math>\int^e_1{\frac{dx}{x}}=\int^e_1{1 \times \frac{1}{x}} \times dx</math>

En intégrant par partie : on obtient <math>\int^e_1{\frac{dx}{x}}=\left[ \frac{x}{x} \right]^e_1 - \int^e_1{x \times \frac{-1}{x^2}} \times dx=1+\int^e_1{\frac{dx}{x}}</math>


  • Au final on a donc : <math>\int^e_1{\frac{dx}{x}}=1+\int^e_1{\frac{dx}{x}}</math> soit 1=1+1=2



x=1 est solution de <math>x^2+x+1=0</math>

  • <math>x^2+x+1=0</math>      (1)
  • x=0 n'est pas solution, donc : <math>x^2+x=-1 \Longleftrightarrow x+1=- \frac{1}{x}</math>       (2)
  • (1) <math>\Longleftrightarrow x+1=-x^2</math>       (3)
  • (2) et (3) <math>\Longrightarrow 1 = x^3</math>       (4)
  • x=1 est solution de (4) donc de (1) ; mais ce résultat est faux car x=1 n'est pas solution de (1)



0>0

  • Soit la suite <math>u=1+\frac{1}{3}+\frac{1}{5}+\frac{1}{7}+\frac{1}{9}+...</math>


  • Soit la suite <math>v=\frac{1}{2}+\frac{1}{4}+\frac{1}{6}+\frac{1}{8}+\frac{1}{10}</math>


  • On a donc <math>2v = 1 + \frac{1}{2} + \frac{1}{3} + \frac{1}{4} + \frac{1}{5} + \frac{1}{6} + ...</math>


  • On remarque que <math>2v=u+v</math> soit <math>u-v=0</math>


  • Or si on calcule u-v : <math>u-v=\underbrace{\left( 1-\frac{1}{2} \right)}_{>0}+\underbrace{\left( \frac{1}{3}-\frac{1}{4} \right)}_{>0}+\underbrace{\left( \frac{1}{5}-\frac{1}{6} \right)}_{>0}+\underbrace{\left( \frac{1}{7}-\frac{1}{8} \right)}_{>0}+ ...>0</math>


  • Donc on a bien u-v=0 et u-v>0



2=1

  • <math>x^2=x+x+x+x+x+x+x+...+x+x+x+x \,\,\,\,(x\,fois)</math>
  • Dérivons des deux côtés : <math>\frac {\partial x^2}{\partial x}=2x</math> et <math>\frac {\partial (\overbrace{x+x+x+x+x+...+x}^{x\,\,fois})}{\partial x}=\overbrace {1+1+1+1+1+...+1}^{x\,\,fois}=x</math>
  • d'où 2x=x ou 2=1



Si |z|=1, alors z=1

  • Démontrons la proposition (fausse bien sûr) : "N'importe quel nombre complexe de module 1 est égal à 1."
  • Soit z un nombre complexe de module 1 : <math>z=e^{i\theta}</math>
  • On pose <math>t={\theta}{2\pi}</math>
  • On a alors : <math>z=e^{2i\pi t}=\left( e^{2i\pi} \right)^t=1^t=1</math>



2=1

  • a=b
  • a2=ba
  • a2-b2=ba-b2
  • (a-b)(a+b)=b(a-b)
  • a+b=b
  • 2a=a
  • 2=1



cos θ>1

  • Soit θ tel que 0<θ<π/2
  • cos θ=cosθ
  • ln(cos θ)=ln(cos θ)
  • 2ln(cos θ)>ln(cos θ)
  • cos2θ>cos θ
  • cos θ>1



1=0

  • Soit a une constante
  • Si x=a, alors on a : <math>\frac{dx}{dx}=\frac{da}{dx}</math> et donc 1=0



Si <math>n \in \mathbb{N}^+</math> alors <math>n=1</math>

  • Soit <math>n \in \mathbb{N}^+</math>
  • On sait que <math>1+2+3+4+...+n=\frac{n(n+1)}{2}</math>
  • Démontrons par récurrence que tout entier strictement positif est égal à 1 :
  • Pour n=1, le théorème est vérifié
  • Supposons donc le théorème vrai pour tout n
  • Pour n>1, alors (n-1) est un entier >0, on peut écrire :
  • <math>1+2+3+4+...+(n-1)=\frac{(n-1)n}{2}</math>
  • Si on ajoute 1 des 2 côtés <math>1+2+3+4+...+(n-1)+n=\frac{(n-1)n}{2}+1</math>
  • Or, <math>1+2+3+4+...+(n-1)+n=1+2+3+4+...+n=\frac{n(n+1)}{2}+1</math>
  • Donc <math>\frac{n(n+1)}{2}=\frac{(n-1)n}{2}+1</math>
  • n(n+1)=n(n-1)+2
  • n2+n=n2-n+2
  • donc 2n=2
  • et enfin n=1